LSAT and Law School Admissions Forum

Get expert LSAT preparation and law school admissions advice from PowerScore Test Preparation.

User avatar
 simonsap
  • Posts: 34
  • Joined: Jun 14, 2021
|
#88008
x6 increase in funding (x3 when we account for inflation)
x2 growth in area that needs funding
cc: more funding is needed, it's not enough

A - money has been wasted; catchy, but that doesn't explain how more money fixes the problem.. it will be wasted again
B - salaries have increased at a rate that consumes the increase in funding in a way that it now makes adequate funding inadequate?
C - cool story, but irrelevant
D - again, cool story, but irrelevant
E - funding was very little, multiplying a small amount by x3 or x6 does not give us a lot; seems legit
User avatar
 Christmaspuppy
  • Posts: 24
  • Joined: Dec 31, 2021
|
#93544
Why answer A is incorrect? If those agency was using the fund in a wrong way, the fund couldn't be enough.
 Rachael Wilkenfeld
PowerScore Staff
  • PowerScore Staff
  • Posts: 1358
  • Joined: Dec 15, 2011
|
#93561
Christmaspuppy, answer choice (A) is a consistent fact between the past and the future. So if the agency has been consistently mismanaged, it's not clear how extra funding would help. Giving more funding to something that is consistently mismanaged won't magically make the extra funds used correctly. It's just giving them more funds to misuse. It doesn't give us anything to explain why we should augment the amount of money. It's just going to throw good money after bad. Answer choice (A) would be a great reason for us to change the management, not add additional funds.

When we are looking to resolve a paradox we are looking to explain how the two seemingly contradictory facts can work together. We start by looking at the two specific facts that seem to not work together. Here, it's that funding has increased sixfold to cover an area that has only increased twofold. Despite this increase, the surprising fact is that we STILL need more money to protect the wetlands, and the government should provide that money. We want to explain how that could make sense. We need something that shows that the current funding is still inadequate despite seeming to be a relatively meaningful increase.

That's what our correct answer choice does. It says that we are increasing such a small amount that we still need more funds to tackle the problem. If we were starting with a tiny sum, increasing it sixfold wouldn't make a dent in a large need. Let's say you have a loan from your buddy. You borrow 10,000 dollars. At first, you pay 1 dollar a month. 10 years later, interest has ballooned the loan to 20,000, but don't worry, you are paying 6 times as much (6 dollars a month). You still need to increase that amount significantly to make a dent in the debt. That's the situation described in the correct answer choice.

Hope that helps.

Get the most out of your LSAT Prep Plus subscription.

Analyze and track your performance with our Testing and Analytics Package.